The mean absolute deviation measures which of the following from the data given?
Average of center or
variability?

Answers

Answer 1

Answer:

Variability

Step-by-step explanation:

Mean absolute deviation (MAD) of a data set is the average distance between each data value and the mean. Mean absolute deviation is a way to describe variation in a data set. Mean absolute deviation helps us get a sense of how "spread out" the values in a data set are.


Related Questions

Answer the question in the picture if you need any questions let me know!

Answers

Answer:

Am I too late too answer?

I need help please this is due today

Answers

Answer:

...

Step-by-step explanation:

I can't really see well there

find the area of a rectangle that measures 6yd by 16yd

a) 44yd^2

b) 22yd^2

c) 48yd^2

d) 96yd^2

Answers

Answer:
The answer is D.

Step-by-step explanation:
Sixteen times six equals ninety-six.
Length times Width equals Area.



(Pls dont delete this again, im trying to help. )

There are four kinds of road signs in a certain town. The number of each kind is shown. What is the ratio of the number of school signs to the number of stop signs?

Answers

Answer:

9:10

Step-by-step explanation:

Could you describe the expression 2(3+4) as a product of two factors? Explain. Can someone pls give me a simple explanation on if the answer is yes or no and why

Answers

Answer:

Distributive Property

Step-by-step explanation:

According to the rule of distributive property, 2(3+4) would be 14.

you want to use the distributive property 2 times 3 = 6 and 2 times 4=8

1. A marine biologist has historical records to show that the chance of
finding shrimp in a catch of ocean animals is 20 percent. The simulation
below models the experimental probability of finding shrimp in at least
one of the next 5 catches. The numbers 1 and 2 represent catches with
shrimp. The numbers 3–10 represent catches without shrimp.
a. What does the marine biologist do?

Answers

Answer:

1,7,3,2,7,10,1

Step-by-step explanation:

I hope you found this useful.

volume with fractions finale 50 POINTS!!!!!

Answers

Answer:

The answer is 1.

Step-by-step explanation:

look at the pic to see how I got it.

Answer:

1

Step-by-step explanation:

Equations to remember:

Length · Width · Height = Volume

LWH = V

---------------------------------------------------

Length = 5/4

Width = 4/3

Height = 3/5

Your equation will look something like this:

[tex]\frac{5}{4} * \frac{4}{3} * \frac{3}{5} = V[/tex]

or

[tex]\frac{5*4*3}{4*3*5} = V[/tex]

----------------------------------------------------

The simple thing about this problem is that, no matter what order they're in, you're multiplying the same numbers on the top and the bottom. Whatever the top is (5*4*3), the bottom will be (4*3*5). If the top and bottom are equal, that means that you'll have a whole number of 1.

----------------------------------------------------

NOMINATORS

[tex]5 * 4 = 20\\20 * 3 = 60[/tex]

So the top will be 60.

----------------------------------------------------

DENOMINATORS

[tex]4 * 3 = 12\\12 * 5 = 60[/tex]

The bottom will also be 60.

-----------------------------------------------------

[tex]\frac{60}{60} = \frac{1}{1} = 1[/tex]

So your answer will be [tex]1cm^{3}[/tex]!

I hope this helped!

What is the length of line segment CD?
Enter your answer, as a decimal rounded to the nearest tenth, in the box.
CD =____units

Answers

Answer:  CD: 8.5

Step-by-step explanation:6^2 = 36

6^2 = 36

36 + 36 = 72

square root 72 = 8.48

so if we round to nearest tenth

CD = 8.5

What is the domain of f(x)=√x-2

Answers

Answer:

The domain of f(x)=√x-2 is  x < -√3 , x > √ 3

Brainliest pls

This is the domain hopefully it helps

Which inequality does this graph show?

Answers

Answer:

D

Step-by-step explanation:

We can find two clear points. (2.5, -0.5) and (4, 0). The slope is 1/3. So our equation is y=(1/3)x + ?. We can figure out the y-intercept by inputting a point. For example, if we input (4,0), we have 0=(4/3)x + ?. So our y-intercept is -4/3. Our equation is y=(1/3)x-4/3. Rearrange and we have y-(1/3)x=-4/3. We multiply 3 to get rid of fractions. 3y-x=-4. Therefore the equation of the line is  3y-x=-4. Now for the inequality part: The shading is BELOW and the line is bolded (not dotted) so it would be "less than or equal to". So, the inequality of the graph would be 3y-x≤-4.                      

Short answer: D
Welcome

Laura started a race at 6:39 PM and finished it at 7:15 PM.
How long did it take her? Give your answer in minutes.

Answers

Answer:

36 mins

Step-by-step explanation:

7:15 - 6:39 = 36 mins

She finished the race in 36 mins.

Have a good day/night

Answer:

36 minutes

Step-by-step explanation:

You subtract the time she started from the time she finished.

So, it would be 7:15 - 6:39.

It would be easier to find the number that, when added to 6:39, results to 7:00. 

To do so, you subtract 39 from 60.

60 - 39 = 21 minutes

Now, add 21 to 15.

21 + 15 = 36 minutes

Therefore, it took her 36 minutes to complete the race.

Hope this helps!

Radiation transfers thermal energy through _________
Which accurately completes the statement?

waves
direct contact
particles
liquid movement

Answers

Answer:

It is Waves, or option, 1.

Step-by-step explanation:

Good luck!

Answer:

The answer is A

Step-by-step explanation:

Good Luck

1 minus what equals 0.863

Answers

Answer:

0.137

Step-by-step explanation:

Answer:

0.137

Step-by-step explanation:

I hope this helps! :)

True 20 words per minute is a good estimate for how fast, in general, the students typed at the beginning of the course. False​

Answers

Answer:

20 words per minute is a good estimate for how fast, in general, the students typed at the beginning of the course. so thats true

Step-by-step explanation:

To rent a certain meeting room, a college charges a reservation fee of $31 and an additional fee of $5.70 per hour. The history club wants to spend less than $82.30 on renting the meeting room.
What are the possible amounts of time for which they could rent the meeting room?
Use for the number of hours the meeting room is rented, and solve your inequality for .

Answers

Answer:

x [tex]\leq[/tex] 9

Step-by-step explanation:

31 + 5.70x [tex]\leq[/tex] 82.30

subtract 82.30 by 31

5.70x [tex]\leq[/tex] 51.3

divide both sides by 5.70

x [tex]\leq[/tex] 9


z > 34. Which of the following statements is the best way to describe the value of z?

Answers

Answer:

The value of z is more than 34.

the value of z is more than 34!

help me you can get 10p

Answers

Answer:

.45 m

Step-by-step explanation:

6.3/14= x

x= .45

Answer is : .45

Explination: 6.3 divided by 4 is 0.45

Ravi started an assignment at 8:25 AM and finished it at 9:53 AM.
How long did it take him? Give your answer in minutes.

Answers

Answer:

it took him and hour, and 28 mins

Step-by-step explanation:

The answer is 88 minutes / 1 hour and 28 minutes

find the perimeter of the square

Answers

Answer:

48

Step-by-step explanation:

12+12+12+12 to each side

48!
You add each side, 12+12+12+12, which should get you 48:)
Hope this helps!

a movie theater offers a reward program that charges a yearly membership fee and a discounted rate per movie ticket. The total cost for a reward program member to see 5 movies is $40.00 and a total costs for 12 movies is $75.00. Assume the jkrelationshup is l near write the equation of the function in the form y=mx+b when x represents the number of movies and y represents the total cost.

Answers

Answer:

— The total cost for a reward program member to see 5 movies is $40 and the total cost for 12 movies is $75. Assume the relationship

Step-by-step explanation:

The equation of the function is y = 5x+5, where x represents the number of movies and y represents the total cost.

What is an equation?

An equation is a mathematical statement that shows that two mathematical expressions are equal.

Given that, The total cost for a reward program member to see 5 movies is $40.00 and a total costs for 12 movies is $75.00.

The coordinates of the equation can be (5, 40) and (12, 75)

The equation of a line passing from two points is given by,

y-y₁ = (y₂-y₁)/(x₂-x₁)(x-x₁)

y-40 = (75-40)/(12-5)(x-5)

y-40 = 35/7(x-5)

y = 5x-25+40

y = 5x+15

Hence, the required equation is y = 5x+15

For more references on equations, click;

https://brainly.com/question/29657988

#SPJ1

ok i genualy need yalls help please

Answers

Part A

Answers:

To find the area of the unshaded region, subtract the area of the  shaded region   from the sum of the areas of the   rectangle   and   semicircle  

----------------

Explanation:

When it comes to area problems, think of a grid of squares. Let's say we had a 10 by 10 grid of tiny squares (10*10 = 100 total). Now let's say 8 of them are shaded in. There are 100 - 8 = 92 unshaded squares to represent the total area of the unshaded region.

That's effectively what's going on with this problem as well. Sure things get a bit tricky with that curved portion on top, but we can still get an approximation if we use smaller and smaller grid sizes. I recommend using graph paper or graphing software that provides a grid.

========================================================

Part B

Answer:  811

----------------

Explanation:

For now, I'm going to ignore the shaded region. I'll come back to it later.

The rectangle is 15 ft by 50 ft. It has an area of length*width = 15*50 = 750 sq ft.

The semicircle up top has an area of [tex]0.5*\pi r^2 = 0.5*\pi*(7.5)^2 \approx 88.3573[/tex]  sq ft approximately. The 7.5 is from dividing the diameter in half to get the radius. As the instructions stated, I used the calculator's version of pi.

The rectangle and semicircle have their areas add to

750+88.3573 = 838.3573

Now we can finally address the shaded region. It is a 3 ft by 9 ft rectangle because "the length is 3 times the width" and the width is 3 ft. This shaded rectangle has area 3*9 = 27 sq ft.

The unshaded region is:

unshaded = total - shaded = 838.3573 - 27 = 811.3573

This rounds to the final answer of 811

Please help!
3x^2 + 1/2 x 7y (When y = 3)

Answers

Factoring the equation:

3x^2 + 1/2 x 7(3)

3x^2 + 1/2 x 21

3 (x^2 + 1/2 x 21)

3 (x^2 + 7/2)

3 x 1/2 (2x^2 +7)

3/2 (2x^2 + 7)

Answer:

3x^2+21/2

Step-by-step explanation:

3x^2+1/2*7(3)

(a)=a so (3)=3

3x^2+1/2*7*3

1/2*7*3=21/2

3x^2+21/2

Hope this helps! Pls give brainliest!

An event at a certain location is described below.

Tectonic plates move towards each other and force rocks upwards into a series of folds.

Which landform will be created at the location?

Mountain range
Ocean basin
Valley
Volcano

Answers

Answer:

Mountain range

Step-by-step explanation:

A mountain range will be created because it says that the tectonic plates "force rocks upwards into a series of folds". The series of folds are the mountain range.

Also: Please mark your questions under the correct category. This is obviously a science question, not a mathematics question.

Answer:

Mountain range

your welcome :)

Please help me will give brainliest to the person with a good and right answer

Answers

Answer:

x= 1.37 or -1.62

Step-by-step explanation:

hope it helps

I believe that is the correct answer to the problem

Divide.

1.156÷10 6

Enter your answer in the box.

Answers

Answer:

1.156 x 10^-6

Step-by-step explanation:

10^6 = 10x10x10x10x10x10 = 1000000

1.156 ÷ 1000000 = 0.000001156

if you simplify that the answer is 1.156 x 10^-6

1.156 x 10^-6 is the answer

A meteorologist recorded the rainfall in Oakdale in two consecutive months. In the first month, there were 1 6/12 inches of rain. In the second month, there were 1 inch of rain. What was the total amount of rainfall during the two months?

Answers

Answer:

There were 2 1/2 inches of rain in 2 months.

Step-by-step explanation:

1. Reduce 1 6/12 to 1 1/2.

2. Add 1 1/2 and 1 (if it is easier for you to add decimals you can reduce the 1 1/2 to 1.5 by dividing 1 by 2. In that case you would get 2.5 as your final answer). The answer is 2 1/2.

Darren and his friend save money for vacation Darren start with $120 in savings each week Darren add $40 to savings Darren’s friend Also begins with $120 in savings but saves at a faster rate than Darren create an equation that could represent the amount of money Y in dollars that Darren’s friend sees vacation after X weeks

Answers

Define the variables:

y= amt. of $ saved by Darren's friend

x= # of weeks

Darren adds $40 to his savings EVERY WEEK. He also started with $120, so the equation is:

y=40x+120

Find the Least common multiple of 4xy cubed and 2x squared y cubed

Answers

Answer
The LCM for 4xy², 2x²y3 is the numeric part 4 multiplied by the variable part.
4xy²
Step-by-step explanation:

Gabe goes to the mall. If b is the number of items he ​bought, the expression 15.41b+28 gives the amount he spent in dollars at one store. Then he spent 27 dollars at another store. Find the expression which represents the amount Gabe spent at the mall. Then estimate how much Gabe spent if he bought 2 items.

Answers

Answer:

Gabe spent about $86 if he bought 2 items.

Step-by-step explanation:

First store: 15.41b+28? Second store: $27b = number of items he bought

15.41b+28+27 = $?

15.41b+55 = $?

15.41(2)+55 = $?

30.82+55 = $?

$85.82

Gabe spent about $86 if he bought 2 items.

Hope this helps!

Answer:

Gabe goes to the mall. If b is the number of items he ​bought, the expression 15.41b+28 gives the amount he spent in dollars at one store. Then he spent 27 dollars at another store. Find the expression which represents the amount Gabe spent at the mall. Then estimate how much Gabe spent if he bought 2 items.

mAngle mean in geometry ?

Answers

Answer:

In geometry, an angle can be defined as the figure formed by two rays meeting at a common end point. An angle is represented by the symbol ∠.

Step-by-step explanation:

It means measurement of an angle
Other Questions
The town of Ferndale has four candidates running for mayor: the town barber, Darrell; the fire chief, Clough the grocer, Abel; and ahomemaker, Belle. A poll of 1000 of the voters shows the following results:DABC = 225CABD = 190CADB = 210BADC = 375Is there a winner using the plurality method? hello humans ~[tex] \\ [/tex]explain how eyes get used to darkness after some time? please help me answer this asap thank you find the value of x giving brainliest to the first right answer!! please help!! Find the value of d that makes the equation true:d - 8 = 5.d = 11d = 12d = 13d = 10 Select whether to use the Pythagorean Theorem or its converse in each situation.Pythagorean TheoremConverseAnight triangle has sidesof length 3 cm and 4 cm.What is the length of itshypotenuse?A triangle has sides oflength 10 m, 15 m, and 8 m,Is it a right triangle?A square has sides of length2 om. What is the length ofits diagonal?A right triangle has one sidethat is 15 ft. Its hypotenuseis 17 ft. How long is theother side? PLEASE HELP ME!!!!!!!!!!! VE ()) You roll a 6-sided die two times. 1) What is the probability of rolling a number greater than 3 and then rolling a number greater than 3? ()) Write your answer as a percentage. % Submit a knife with a sharp edge that has saw like notches or teeth use to slice bread< fruit and vegetable Complete each sentence with the present perfect form of the appropriate verb. Use eachverb only once.abrirdecirdescubrirescribirmorirromperver6. Cristbal se____un brazo.7. T y yo_____un accidente.8. Los arquelogos____tumbas en Egipto.9. Yo le__una carta a mi prima.10. El seor Lpez_____de un ataque al corazn.11. Quin_______la puerta?12. Mis padres me_______que no tengo que estudiar esta tarde. where is pakistan what is its cultural value? Whats the value of z I need help finding the domain and range!! -Algebra 7 > 2 - 4 all possible solutions 9. How do minerals make it to the surface of the earth's crust? please help me!! Combustion analysis-empirical formulaI'm having trouble understanding the concept could someone help me to understand how to solve a question like this? HELP ASAP!!Which New Deal agency built state parks in Texas? AAA CCC FERA SSA Select the correct answer.Which of the following is an example of using effective speaking skills?ABC D please help oooooooo Q11 The price of a smartphone is $600 and a smart watch is $250. If you buy them together you get a discount of 20%. If you decide to buy both, how much will you end up paying?Q12Another store is offering the smart watch for $100 if purchased with the smartphone. Is this a better deal than the previous one (Q11)? How can you change grams into kilograms